An urn contains7green and 8red balls. Five balls are randomly drawn from the urn in succession, with replacement. That is, after each draw, the selected ball is returned to the urn. What is the probability that all 5 balls drawn from the urn are red? Round your answer to three decimal places

Answers

Answer 1

The probability that all 5 balls drawn from the urn are red is 0.069 (approximately).

The number of balls in the urn = 7 green + 8 red = 15 ballsThere are 5 balls drawn from the urn in succession, with replacement. That is, after each draw, the selected ball is returned to the urn. Thus, the probability that each of the 5 balls drawn is red can be found as follows;Probability of drawing a red ball on any draw = 8/15Probability of drawing 5 red balls = P(Red, Red, Red, Red, Red)= (8/15) * (8/15) * (8/15) * (8/15) * (8/15)= (8/15)^5= 0.0693Rounding to three decimal places.

The probability of drawing all 5 balls red is 0.069. Therefore, the probability that all 5 balls drawn from the urn are red is 0.069 (approximately).

Learn more about Succession here,

https://brainly.com/question/1824935

#SPJ11


Related Questions

plz I NEEEEED HELP NOWWWW PLZ PLZ PLZ

Answers

○=> Correct option :[tex]\color{hotpink}\bold{ (a) \: 130}[/tex]○=> Steps to derive correct option :

Angle (p+7)° and angle (3p+1)° are a linear pair so their sum will be equal to 180°.

Which means :

[tex] =\tt 3p + 1 + p + 7 = 180[/tex]

Let us solve that equation to find the value of p and the two angles :

[tex] =\tt 3p + 1 + p + 7 = 180[/tex]

[tex] =\tt 3p + p + 1 + 7 = 180[/tex]

[tex] =\tt 4p + 8 = 180[/tex]

[tex] = \tt4p = 180 - 8[/tex]

[tex] = \tt4p = 172[/tex]

[tex] =\tt p = \frac{172}{4} [/tex]

[tex]\hookrightarrow \tt \color{plum}p = 43[/tex]

Thus, value of p = 43

Measure of angle (p+7)° :

[tex] =\tt 43 + 7[/tex]

[tex]\tt\color{plum}angle \: (p + 7)° =\tt 50°[/tex]

Measure of angle (3p+1)° :

[tex] =\tt 43 \times 3 + 1[/tex]

[tex] = \tt129 + 1[/tex]

[tex]\tt\color{plum}angle \: (3p + 1)° = 130°[/tex]

Thus, the measure of the larger angle = 130°

Therefore, the correct option is (a) 130

Check whether -2 is a zero of the polynomial 3x2 + 4x + 32 .

Answers

Answer:

x = - 2 is not a zero

Step-by-step explanation:

If x = - 2 is a zero of the polynomial then f(- 2) = 0

f(x) = 3x² +4x + 32

f(- 2) = 3(- 2)² + 4(- 2) + 32 = 3(4) - 8 + 32 = 12 - 8 + 32 = 36

Since f(- 2) ≠ 0 , then x = - 2 is not a zero of the polynomial

How do the 6s in 0.96 and 93.601 compare?
The value of the 6 in 0.96 is 100 times the value of the 6 in 93.601.
The value of the 6 in 0.96 is 10 times the value of the 6 in 93.601.
The value of the 6 in 0.96 is 110 the value of the 6 in 93.601.
The value of the 6 in 0.96 is 1100 the value of the 6 in 93.601.

Answers

Answer:The value of the 6 in 0.96 is 1/10 the value of the 6 in 93.601. This is because the 6 in 0.9 compared to the 6 in 93.601 which is in the tenths place.

Step-by-step explanation:The value of the 6 in 0.96 is 1/10 the value of the 6 in 93.601. This is because the 6 in 0.96 is in the hundredths place, compared to the 6 in 93.601 which is in the tenths place.

Answer:

The value of the 6 in 0.96 is 110 the value of the 6 in 93.601

Step-by-step explanation:

Find the area of the composite
Figure-someone help plz

Answers

57 in^2

Steps on paper.

Answer:

84

Step-by-step explanation:

please help me please​

Answers

Answer:

2πx - 4

Step-by-step explanation:

C = 2πr

r = (x-2)

C = 2π(x-2)

C = 2πx - 4

In ΔWXY, x = 680 inches, w = 900 inches and ∠W=157°. Find all possible values of ∠X, to the nearest degree.

Answers

Given:

In ΔWXY, x = 680 inches, w = 900 inches and ∠W=157°.

To find:

The all possible values of ∠X, to the nearest degree.

Solution:

Law of Sines:

[tex]\dfrac{a}{\sin A}=\dfrac{b}{\sin B}=\dfrac{c}{\sin C}[/tex]

For ΔWXY,

[tex]\dfrac{w}{\sin W}=\dfrac{x}{\sin X}=\dfrac{y}{\sin Y}[/tex]

Now,

[tex]\dfrac{w}{\sin W}=\dfrac{x}{\sin X}[/tex]

[tex]\dfrac{900}{\sin (157^\circ)}=\dfrac{680}{\sin X}[/tex]

[tex]900\sin X=680\sin (157^\circ)[/tex]

[tex]\sin X=\dfrac{680}{900}\sin (157^\circ)[/tex]

[tex]\sin X=0.295219[/tex]

[tex]X=\sin^{-1}(0.295219)[/tex]

[tex]X=17.17067[/tex]

[tex]X\approx 17[/tex]

Therefore, the value of ∠X is 17 degrees.

I need help, please solve this math problem. If you don't know the answer please don't answer. You will get reported.

Answers

Answer:

I believe it's A 2¹²

Step-by-step explanation:

The answer lies in the question itself.

In the equation, we have

3x - y = 12.

This means 3x = 12 + y.

We have to find 8^x/2^y.

8 can be written as 2^3. Therefore, 8^x = 2^3x.

But 3x = 12 + y, hence can be replaced.

Now we have, in the fraction, 2^(12 + y)/2^y.

This can be written as 2^(12 + y - y) = 2^12.

Tyrone has 16 model airplanes on his shelf. He has 4 times as many model airplanes as model trains. How many model trains does Tyrone have?


Write multiplication and division equations to model and solve the problem. Use m for the unknown

Answers

Step-by-step explanation:

Write multiplication and division equations to model and solve the problem. Use m for the unknown

Help me solve thank you will give 20 points

Answers

Answer:

Natalie bought 500 apples at $0.40 each, then she pays $0.40 500 times, this means that the total cost of the 500 apples is:

Cost = 500*$0.40 = $200

Now she threw away n apples from the 500 apples, then the number of apples that she has now is:

apples = 500 - n

And she sells the remaining apples for $0.70 each.

a) The amount that she gets by selling the apples is:

Revenue = (500 - n)*$0.70

b) We know that she did not make a loss, then the revenue must be larger than the cost, this means that:

cost ≤ revenue

$200 ≤ (500 - n)*$0.70

c) We need to solve the inequality for n.

$200 ≤ (500 - n)*$0.70

$200/$0.70 ≤ (500 - n)

285.7  ≤ 500 - n

n + 285.7 ≤ 500

n ≤ 500 - 285.7

n ≤ 214.3

Then the maximum value of n must be equal or smaller than 214.3

And n is a whole number, then we can conclude that the maximum number of rotten apples can be 214.

PLEASE HELP! DUE SOON!

Answers

It’s 1.5. The question asking for the amount of seconds (x axis) have passed when the bird’s height (y axis) equals 0. Basically the x value of the point where the line touched the x axis

(GIVING BRAINILEST) Which characteristic do stable ecosystems tend to have s

Answers

Answer:

It will be C

Step-by-step explanation:

This is because ecosystems worh high biodiversity tend to be more stable

Hope this Helped

Answer:

The awnser is c i took the test ( a high amount of bio deversety)

Step-by-step explanation:

–4v – 9z – 9v + 2 – 4v find the sum

Answers

Answer:

-1x(13v+9z+2)

Step-by-step explanation: try solve this :)

Answer

-17v-9z+2

Step-by-step explanation:

First collect the like terms

-4v-4v-9v-9z+2

-17v-9z+2

Since we can no longer add up due to they are unlike term. Therefore our answer is -17v-9z+2

Find the surface area of the square pyramid shown in the net below. The surface area of the
figure is
cm. (Round your answer to the nearest tenth.)
4 cm
8.4 cm

Answers

Answer:

93 cm

Step-by-step explanation:

there's 4 that describe the same function

Answers

Answer:

A,B,C,E

Step-by-step explanation:

for a everything is multiplied by 10

for c everything is divided by 6

for b everything is divided by 12

e is the only other one that fits

for a everything is multiplied by 10

for c everything is divided by 6

for b everything is divided by 12

e is the only other one that fits

ABCE is the answer to the question

can someone please help? i’ll mark brainliest!!

Answers

Answer:

z+5 z+2 Z i think

Step-by-step explanation:

the lenght is longer to shorter

Answer:

angle # > angle * > angle $

Step-by-step explanation:

Since z + 5 > z+ 1 > z

So, angle # > angle * > angle $ (Angle opposite to greater side is greater and opposite to smallest side is smallest)

What is the volume of the rectangular prism below?​

Answers

Answer:

V=140

Step-by-step explanation:

5*4=20

20*7=140

Please solve this question

Answers

B: 12/5 because when looking at the sin in sohcahtoa you want to look at the opposite and adjacent side the the angle X.. the adjacent is right next to the angle the hypotenuse is always the slanted side and the opposite is whatever is left Ig

d) What would be the total cost of purchasing the number of shirts needed to use your coupon—after your coupon is applied and a 7.5% sales tax is charged on the purchase?

Answers

Answer: Multiply 7.5% by the price after subtracting the coupon's value.

Step-by-step explanation:

Let's assume that the cost of the shirts is x. So the coupon would subtract y from x so that's x-y which gives us z. Then to find z, on a calculator just put in 7.5% x z. I used variables since I don't know the value of the coupon or the original price of the shirts.

The sides of a rectangle have length x+1 and width x-5. which equation below describes the area A of the rectangle in terms of x?​

Answers

Option C

Area = length * width
Area = (x + 1)(x -5)
Use FOIL

x*x + x*-5 + 1*x + 1*-5
x^2 - 5x + x - 5
x^2 - 4x - 5

A student claims that x=3 and 2+3i are the zeros of a quadratic function. Explain why this is incorrect

Answers

Answer:

Because it's not a quadratic function.

Step-by-step explanation:

The imaginary roots of a quadratic equation will always be in form of a ± b thus 3 and 2 + 3i will be incorrect.

What are the roots of an equation?

The roots of an equation are the solution of that equation, since an equation consists of hidden values of the variable so to determine them by different processes and then the resultant is called roots.

The standard form of a quadratic equation is,

ax² + bx + c = 0

The root of it is given by,

x = [-b ± √(b²-4ac)]/2a

For being this as imaginary the term b²-4ac < 0

Let's say b²-4ac = -m then

x = (-b/2a) ± (m/2a)i so both root will be imaginary.

Given that one root is real (3) and the second is imaginary (2 + 3i) is not possible.

Hence "The imaginary roots of a quadratic equation will always be in form of a ± b thus 3 and 2 + 3i will be incorrect".

To learn more about the roots of equations,

https://brainly.com/question/12029673

#SPJ2

i need help!!!! list the factors of -108 that equal to -3

Answers

Answer:

9 and -12

Step-by-step explanation:

This is the answer 9 and -12

PLEASE HELP ASAP!!
Graph the function
h (x) = -4x-1.

Answers

Answer:

Hope this helps!

A 1,000 mL solution of acetic acid contains 26 mL of pure acetic acid. Find the percent concentration of this solution.

Answers

Given:

Total solution : 1000mL

Pure acetic acid = 26mL

To find:

The percent concentration of given solution.

Solution:

We know that,

[tex]\text{Percent concentration}=\dfrac{\text{Pure acetic acid}}{\text{Total solution}}\times 100[/tex]

Putting the given values, we get

[tex]\text{Percent concentration}=\dfrac{26}{1000}\times 100[/tex]

[tex]\text{Percent concentration}=\dfrac{26}{10}[/tex]

[tex]\text{Percent concentration}=2.6[/tex]

Therefore, the percent concentration of this solution is 2.6%.

I need help, I don't just need to answer I need to know the steps on how to solve it asap

Answers

Your goal when solving problems like these are to get V by itself. Meaning, on one side of the equal sign. So first you are to add -2 over to the side with the 4. Then multiple that by 11. To undo the division

what is the sum of 17 and a difference of 5

Answers

Answer:

The answer is 12.

~Hoped this helped~

Answer:

please mark me as brainliest answer

Step-by-step explanation:

The sum of two numbers is 17 and their difference is 5. What are the two numbers? Let's start by calling the two numbers we are looking for x and y.

The sum of x and y is 17. In other words, x plus y equals 17 and can be written as equation A:

x + y = 17

The difference between x and y is 5. In other words, x minus y equals 5 and can be written as equation B:

x - y = 5

Now solve equation B for x to get the revised equation B:

x - y = 5

x = 5 + y

Then substitute x in equation A from the revised equation B and then solve for y:

x + y = 17

5 + y + y = 17

5 + 2y = 17

2y = 12

y = 6

Now we know y is 6. Which means that we can substitute y for 6 in equation A and solve for x:

x + y = 17

x + 6 = 17

X = 11

Summary: The sum of two numbers is 17 and their difference is 5. What are the two numbers? Answer: 11 and 6 as proven here:

Sum: 11 + 6 = 17

Difference: 11 - 6 = 5

Idris bought 9 tomato plants for $33.12. How much did each tomato plant cost?

Answers

33.12 ÷ 9 = 3.68$3.68

Answer:

9 tomat plant cost= 33.12

each tomato plant cost = 33.12 / 9

answer = 3.68 dalours

−12 = −0.6k solve for k

Answers

Answer:

-20

Step-by-step explanation:

Step-by-step explanation:

-12=-0.6k

divide both sides by -0.6

k=20

lmk if this is right

The product of two numbers is 24. One of the numbers is 10 units less than the other. The numbers are:
A. 0 and 12
B. 12 and -2
C. 10 and 14
D. -12 and 2
E. -12 and -2
Which is true?

Please I need your help fast ​

Answers

Answer:

E. -12 and -2

Step-by-step explanation:

-12 x -2 = 24

brainliest please

Answer:

Obviously -12 and -2 is true.

Find an ordered triple that represents 65 = (2, 6, 8) and Ž = (1, -2, 6).

Answers

Answer:

< 40, 28, 72 >

Step-by-step explanation:

Given

y = < 2, 6, 8 > and z = < 7, - 2, 6 > , then

6y + 4z

= 6 < 2, 6, 8 > + 4 < 7, - 2, 6 >

Multiply each component by the scalar quantity

= < 6(2), 6(6), 6(8) > + < 4(7), 4(- 2), 4(6) >

= < 12, 36, 48 > + < 28, - 8, 24 >

Add corresponding components

= < 12 + 28, 36 - 8, 48 + 24 >

= < 40, 28, 72 >

Write Each Ratio as a Percent

1. 14 losses in 50 games

2. 4/5 of the students ride the bus

3. if AB represents 25% what is the length of a line segment that is 100%?

A________________B
5ft

Answers

Answer:

28%

80%

20

Step-by-step explanation:

hope this helps

Other Questions
stub-outs should extend ____ through the finished wall. if you plug an electric toaster rated at 110v into a 220v outlet the current drawn by the toaster will be Given a set of returns, the wider the distribution of those returns, theSelect one:A. lower the average rate of return.B. lower the variance.C. lower the volatility.D. higher the number of those returns.E. higher the standard deviation. This is due today! So please answer this fast I have to chance up on other assignments A process in which steps are revisited and the flow is more like a circle than aline.summarizesynthesizecyclicalannotate let be a linear transformation defined by mapping every to av bw . find a matrix such that . an international company has employees with complementary skill sets work together toward a mutual goal. what is the organizational structure of this company? The age of juveniles appearing before the juvenile court varies greatly from state to state, with the minimum being:a. 6b. 11c. 14d. 20 Identify each substance as an acid or a base. Liquid drain cleaner, pH 13. 5 milk, pH 6. 6. reviewing a document or web site is the same thing as testing it.T/F? a sample of helium gas occupies 19.1 l at 23c and 0.956 atm. what volume will it occupy at 40c and 1.20 atm? [1] (a) The angular size of the Crab SNR is 42 and its distance from Earth is approximately 2000pc (see Fig. 4). Estimate the linear dimensions of the nebula.(b) Using the measured expansion rate of the Crab and ignoring any accelerations since the time of the supernova explosion, estimate the age of the nebula. ______ is an increasingly popular method of specifically treating organic waste such as food and yard waste. How do you distinguish between the binge-eating or purging type of anorexia nervosa and bulimia nervosa, purging type?a. The bulimic type involves throwing up, and the anorexic type involves fasting.b. Altered eating and exercise habits result in missed periods in the bulimic type only.c. The bulimic type results in more severe health consequences than the anorexic type.d. People with the bulimic type are normal weight, people with the anorexic type are underweight. The city of MonteAlbn includedwhich of thefollowing?A. a gold palaceB. large cathedralsC. colossal stone headsD. a star-gazing observatory Freddie has a bag with 7 blue counters, 8 yellow counters and 15 black counters When illuminated by red light of frequency f = 6 x 1014 Hz, what is the stopping voltage of a photocell, made of a metal plate with a work function W = 2 eV?a) 3.5 Vb) 1.5 Vc) 2.5 Vd) 0.5 V a 28-year-old woman presents to her primary care provider complaining of abdominal bloating, cramping and diarrhea most days of the week for the last four months. the diarrhea commonly occurs in the morning or following meals. episodes occur four or five times per day and are of moderate volume. she denies fever, nausea, vomiting, periods of constipation, or change in appetite. the diarrhea does not wake her from her sleep and she reports no blood or pus in her stool. she states the periods of diarrhea will last several months and then seem to resolve without explanation. in between diarrhea episodes, she has normal formed bowel movements once per day. a review of her symptom diary reveals no food associations. she denies any recent travel. what is the most likely diagnosis? communities implementing case-finding measures and providing adequate health personnel, equipment, and facilities for the community are examples of: the climate of the earth has changed dramatically several times over its long history. which of these was not a factor in those changes?a.Changes in the amount of heat coming from the core of the Earth b.Changes in ocean circulation patterns c.Plate tectonics d.Changes in the natural carbon cycle e.Changes in the tilt and orbit of the Earth